Q15

 
mromeo2
Thanks Received: 0
Vinny Gambini
Vinny Gambini
 
Posts: 1
Joined: August 07th, 2013
 
 
 

Q15

by mromeo2 Fri Sep 20, 2013 4:06 pm

I'm having a hard time figuring this out w/o trudging away with hypotheticals for every answer. I can eliminate (D) and (E) but i dont know how to attack the other 3. Advice?
 
christine.defenbaugh
Thanks Received: 585
Atticus Finch
Atticus Finch
 
Posts: 536
Joined: May 17th, 2013
 
 
 

Re: Q15

by christine.defenbaugh Tue Sep 24, 2013 12:09 am

Complete and accurate list questions can be a real challenge! This question type is essentially a 'could be true' question - which group could be true (without any additions) for Zone 3. Since one answer could be true, 4 answers must be false. So we want to be focused on finding the four rule violators and eliminating them!

You are absolutely correct that without a more informative original setup, if this were the only question we were attacking, we'd have little choice but to play out each answer choice a bit to see which ones skewer themselves on rules. You've correctly noted that (D) and (E) can be eliminated fairly quickly.

Rule Violators
(D) puts both T and U in Zone 3, leaving Zone 2 without either of them.
(E) puts both P and U in Zone 3, leaving T torn between fulfulling Zone 1's rule or Zone 2's!

A key at this point is realizing that your remaining three answer choices all have 2 elements in Zone 3. If there are only 2 elements in Zone 3, there must be only 1 element in Zone 2 (as it must have fewer), forcing the remaining 4 elements into Zone 1! So, we have a 4/1/2 setup....wait a second....we've done this before!

We did it for question 13!
Image

The only valid 2 party lists for Zone 3 are SU and KM! Only answer (A) has one of these two lists: K and M!

This is a great example of recognizing how previous work can relate to the question at hand, saving you significant effort.


Another Perspective: Frames
If we had originally set up frames for this game, that could have been a big time saver here as well. These three frames are set up around the P/T and T/U rules. Note that there are only three possibilities, since T appears in both rules.
Image

Rule Violators
(B) T can only be in Zone 3 in Frame 3, which requires M to be in Zone 3 as well.
(C) P and Q can only be in Zone 3 in Frame 1, which requires there be at least one other person in the zone.
(D) T can only be in Zone 3 in Frame 3, which requires S and U to be in Zone 2
(E) P and Q can only be in Zone 3 in Frame 1, which requires S and U to be in Zone 2.

Only (A) is permissible, and only in Frame 2!

There are multiple lines of attack here: framing, using past work, and if all else fails, plugging in the answers to find rule violators.

Please let me know if this answers your question!